Difference between revisions of "2010 AMC 10B Problems/Problem 24"

(Solution 1)
(Redirected page to 2010 AMC 12B Problems/Problem 19)
(Tag: New redirect)
 
(8 intermediate revisions by 2 users not shown)
Line 1: Line 1:
== Problem ==
+
#redirect [[2010 AMC 12B Problems/Problem 19]]
A high school basketball game between the Raiders and Wildcats was tied at the end of the first quarter. The number of points scored by the Raiders in each of the four quarters formed an increasing geometric sequence, and the number of points scored by the Wildcats in each of the four quarters formed an increasing arithmetic sequence. At the end of the fourth quarter, the Raiders had won by one point. Neither team scored more than <math>100</math> points. What was the total number of points scored by the two teams in the first half?
 
 
 
<math>\textbf{(A)}\ 30 \qquad \textbf{(B)}\ 31 \qquad \textbf{(C)}\ 32 \qquad \textbf{(D)}\ 33 \qquad \textbf{(E)}\ 34</math>
 
 
 
== Solution 1 ==
 
 
 
 
 
Let <math>a,ar,ar^{2},ar^{3}</math> be the quarterly scores for the Raiders. We know <math>r > 1</math> because the sequence is said to be increasing. We also know that each of <math>a, ar, ar^2, ar^3</math> is an integer. We start by showing that '''<math>r</math> must also be an integer.'''
 
 
 
Suppose not, and say <math>r = m/n</math> where <math>m>n>1</math>, and <math>\gcd(m,n)=1</math>. Then <math>n, n^2, n^3</math> must all divide <math>a</math> so <math>a=n^3k</math> for some integer <math>k</math>. Then <math>S_R = n^3k + n^2mk + nm^2k + m^3k < 100</math> and we see that even if <math>k=1</math> and <math>n=2</math>, we get <math>m < 4</math>, which means that the only option for <math>r</math> is <math>r=3/2</math>. A quick check shows that even this doesn't work. Thus <math>r</math> must be an integer.
 
 
 
Let <math>a, a+d, a+2d, a+3d</math> be the quarterly scores for the Wildcats. Let <math>S_W = a+(a+d) + (a+2d)+(a+3d) = 4a+6d</math>. Let <math>S_R = a+ar+ar^2+ar^3 = a(1+r)(1+r^2)</math>. Then <math>S_R<100</math> implies that <math>r<5</math>, so <math>r\in \{2, 3, 4\}</math>. The Raiders win by one point, so<cmath>a(1+r)(1+r^2) = 4a+6d+1.</cmath>
 
*If <math>r=4</math> we get <math>85a = 4a+6d+1</math> which means <math>3(27a-2d) = 1</math>, which is absurd.
 
*If <math>r=3</math> we get <math>40a = 4a+6d+1</math> which means <math>6(6a-d) = 1</math>, which is also absurd.
 
*If <math>r=2</math> we get <math>15a = 4a+6d+1</math> which means <math>11a-6d = 1</math>. Reducing modulo 6 we get <math>a \equiv 5\pmod{6}</math>. Since <math>15a<100</math> we get <math>a<7</math>. Thus <math>a=5</math>. It then follows that <math>d=9</math>.
 
Then the quarterly scores for the Raiders are <math>5, 10, 20, 40</math>, and those for the Wildcats are <math>5, 14, 23, 32</math>. Also <math>S_R = 75 = S_W + 1</math>. The total number of points scored by the two teams in the first half is <math>5+10+5+14=\boxed{\textbf{(E)}\ 34}</math>.
 
 
 
'''NOTE''': When checking to make sure that r is an integer, we must also check <math>m = 4</math> <math>n = 3</math> for it to be complete, as those two solutions also fit the conditions
 
 
 
== Solution 2 ==
 
Represent the teams' scores as: <math>(a, an, an^2, an^3)</math> and <math>(a, a+m, a+2m, a+3m)</math>
 
 
 
We have <math>a+an+an^2+an^3=4a+6m+1</math>
 
Factoring out the <math>a</math> from the left side of the equation, we can get <math>a(1+n+n^2+n^3)=4a+6m+1</math>, or <math>a(n^4-1)/(n-1)=4a+6m+1</math>
 
 
 
Since both are increasing sequences, <math>n>1</math>. We can check cases up to <math>n=4</math> because when <math>n=5</math>, we get <math>156a>100</math>. When
 
* <math>n=2, a=[1,6]</math>
 
*<math>        n=3, a=[1,2]</math>
 
*<math>          n=4, a=1</math>
 
Checking each of these cases individually back into the equation <math>a+an+an^2+an^3=4a+6m+1</math>, we see that only when <math>a=5</math> and <math>n=2</math>, we get an integer value for <math>m</math>, which is <math>9</math>. The original question asks for the first half scores summed, so we must find <math>(a)+(an)+(a)+(a+m)=(5)+(10)+(5)+(5+9)=\boxed{\textbf{(E)}\ 34}</math>
 
 
 
*'''Note:''' This solution is incomplete because it doesn't consider fractional values of <math>r</math>.
 
 
 
== Solution 3 ==
 
 
 
*'''Note:''' This solution is incomplete because it doesn't consider fractional values of <math>n</math>.
 
 
 
As above, represent the teams' scores as: <math>(a, an, an^2, an^3)</math> and <math>(a, a+m, a+2m, a+3m)</math>.
 
 
 
Note that the Wildcat's sum, <math>4a+6m</math>, is even. Therefore, since the Wildcat's sum is one less than the Raiders, the Raider's team's score should be odd. But if all of <math>(a, an, an^2, an^3)</math> are of the same parity, the sum will be even. If <math>a</math> is even, then the rest of the scores will be even, so clearly <math>a</math> is odd. Then, <math>n</math> is even.
 
 
 
But if <math>n=4</math>, only <math>a=1</math> satisfies the requirement that the total score of each team is less than <math>100</math>. We can test this out and see it doesn't work.
 
 
 
Therefore, <math>n=2</math>. If we try <math>a=(1,3,5)</math>, we quickly see only <math>a=5</math> satisfies all of the conditions. Therefore, our team's scores are <math>(5,10,20,40)</math> and <math>(5,14,23,32)</math>, and the answer is <math>(5)+(10)+(5)+(14)=\boxed{\textbf{(E)}\ 34}</math>.
 
==Discussion==
 
Couldn't we also say Raiders:1,2,4,8 and Wildcats:1,3,5,7 so the Raiders have won by one point?
 
[s]Is this false because in actual basketball you can only score 2 or more points?[/s]  
 
---> Raiders would have a total of 15 points and the WildCats would have a total of 16 points.
 
 
 
*'''Note:''' In example above, the Raiders score a total of 15 points while the Wildcats score 16. So it is the '''Wildcats''' who win by one point, not the '''Raiders'''.
 
 
 
*'''Note 2:''' It is possible in a game of basketball to score 1 point. A person is fouled and makes the free throw. That is 1 point.
 
 
 
== See also ==
 
{{AMC10 box|year=2010|ab=B|num-b=23|num-a=25}}
 
{{MAA Notice}}
 

Latest revision as of 20:54, 26 May 2020